How strong is the axiom of well-ordered choice?No uncountable ordinals without the axiom of choice?Axiom of choice , Hartogs ordinals, well-ordering principleWhich sets are well-orderable without Axiom of Choice?Well-orderings of $mathbb R$ without Choice“There is no well-ordered uncountable set of real numbers”Axiom of choice and the well ordering principleThe class of well-founded sets satisfies the axiom of foundation and the axiom of choiceAxiom of Choice is equivalent to Well-ordering Theorem: Hrbacek, Jech - “Introduction to Set Theory”the power set of every well-ordered set is well-ordered implies well orderingHow strong are weak choice principles?

Should I apply for my boss's promotion?

Where is the License file location for Identity Server in Sitecore 9.1?

How to write a chaotic neutral protagonist and prevent my readers from thinking they are evil?

A vote on the Brexit backstop

Precision notation for voltmeters

Why would /etc/passwd be used every time someone executes `ls -l` command?

How to educate team mate to take screenshots for bugs with out unwanted stuff

Is it appropriate to ask a former professor to order a library book for me through ILL?

Use Mercury as quenching liquid for swords?

Professor forcing me to attend a conference, I can't afford even with 50% funding

Was this cameo in Captain Marvel computer generated?

Short story about an infectious indestructible metal bar?

How does a sound wave propagate?

ESPP--any reason not to go all in?

Should we avoid writing fiction about historical events without extensive research?

Do I need a return ticket to Canada if I'm a Japanese National?

School performs periodic password audits. Is my password compromised?

Sort array by month and year

I am the person who abides by rules but breaks the rules . Who am I

Averaging over columns while ignoring zero entries

Help! My Character is too much for her story!

Boss Telling direct supervisor I snitched

PTIJ: Sport in the Torah

Will the concrete slab in a partially heated shed conduct a lot of heat to the unconditioned area?



How strong is the axiom of well-ordered choice?


No uncountable ordinals without the axiom of choice?Axiom of choice , Hartogs ordinals, well-ordering principleWhich sets are well-orderable without Axiom of Choice?Well-orderings of $mathbb R$ without Choice“There is no well-ordered uncountable set of real numbers”Axiom of choice and the well ordering principleThe class of well-founded sets satisfies the axiom of foundation and the axiom of choiceAxiom of Choice is equivalent to Well-ordering Theorem: Hrbacek, Jech - “Introduction to Set Theory”the power set of every well-ordered set is well-ordered implies well orderingHow strong are weak choice principles?













10












$begingroup$


I sometimes see references to the "Axiom of Well-Ordered Choice," but I'm not sure how strong it is. It states that every well-ordered family of sets has a choice function.



By "well-ordered family," I don't mean that the sets within the family are well-ordered, but that the family must index all the sets within the family by some ordinal.



How strong is this axiom? Can it prove the Hahn-Banach theorem, the ultrafilter lemma, anything about measurable sets, etc? Does it have any implications about what sets can be well-ordered (the reals for instance), or perhaps prove anything about the Hartogs number of sets, etc?



Does anyone have a reference for this?










share|cite|improve this question











$endgroup$











  • $begingroup$
    ...isn't this axiom a consequence of ZF? One can choose the (well-defined, since it's well-ordered) lexicographically least element of each set in the family...
    $endgroup$
    – Steven Stadnicki
    8 hours ago






  • 2




    $begingroup$
    That is a family of well-ordered sets, not a well-ordered family of (arbitrary sets).
    $endgroup$
    – Mike Battaglia
    8 hours ago










  • $begingroup$
    Ahh, I missed that distinction. Thank you!
    $endgroup$
    – Steven Stadnicki
    8 hours ago










  • $begingroup$
    I've never seen this axiom before. Does the family itself need to be a set or can it be a proper class?
    $endgroup$
    – Robert Shore
    8 hours ago










  • $begingroup$
    Google suggests this: settheory.mathtalks.org/andreas-blass-well-ordered-choice
    $endgroup$
    – Carl Mummert
    8 hours ago















10












$begingroup$


I sometimes see references to the "Axiom of Well-Ordered Choice," but I'm not sure how strong it is. It states that every well-ordered family of sets has a choice function.



By "well-ordered family," I don't mean that the sets within the family are well-ordered, but that the family must index all the sets within the family by some ordinal.



How strong is this axiom? Can it prove the Hahn-Banach theorem, the ultrafilter lemma, anything about measurable sets, etc? Does it have any implications about what sets can be well-ordered (the reals for instance), or perhaps prove anything about the Hartogs number of sets, etc?



Does anyone have a reference for this?










share|cite|improve this question











$endgroup$











  • $begingroup$
    ...isn't this axiom a consequence of ZF? One can choose the (well-defined, since it's well-ordered) lexicographically least element of each set in the family...
    $endgroup$
    – Steven Stadnicki
    8 hours ago






  • 2




    $begingroup$
    That is a family of well-ordered sets, not a well-ordered family of (arbitrary sets).
    $endgroup$
    – Mike Battaglia
    8 hours ago










  • $begingroup$
    Ahh, I missed that distinction. Thank you!
    $endgroup$
    – Steven Stadnicki
    8 hours ago










  • $begingroup$
    I've never seen this axiom before. Does the family itself need to be a set or can it be a proper class?
    $endgroup$
    – Robert Shore
    8 hours ago










  • $begingroup$
    Google suggests this: settheory.mathtalks.org/andreas-blass-well-ordered-choice
    $endgroup$
    – Carl Mummert
    8 hours ago













10












10








10


1



$begingroup$


I sometimes see references to the "Axiom of Well-Ordered Choice," but I'm not sure how strong it is. It states that every well-ordered family of sets has a choice function.



By "well-ordered family," I don't mean that the sets within the family are well-ordered, but that the family must index all the sets within the family by some ordinal.



How strong is this axiom? Can it prove the Hahn-Banach theorem, the ultrafilter lemma, anything about measurable sets, etc? Does it have any implications about what sets can be well-ordered (the reals for instance), or perhaps prove anything about the Hartogs number of sets, etc?



Does anyone have a reference for this?










share|cite|improve this question











$endgroup$




I sometimes see references to the "Axiom of Well-Ordered Choice," but I'm not sure how strong it is. It states that every well-ordered family of sets has a choice function.



By "well-ordered family," I don't mean that the sets within the family are well-ordered, but that the family must index all the sets within the family by some ordinal.



How strong is this axiom? Can it prove the Hahn-Banach theorem, the ultrafilter lemma, anything about measurable sets, etc? Does it have any implications about what sets can be well-ordered (the reals for instance), or perhaps prove anything about the Hartogs number of sets, etc?



Does anyone have a reference for this?







set-theory axiom-of-choice foundations well-orders






share|cite|improve this question















share|cite|improve this question













share|cite|improve this question




share|cite|improve this question








edited 7 hours ago







Mike Battaglia

















asked 8 hours ago









Mike BattagliaMike Battaglia

1,4371127




1,4371127











  • $begingroup$
    ...isn't this axiom a consequence of ZF? One can choose the (well-defined, since it's well-ordered) lexicographically least element of each set in the family...
    $endgroup$
    – Steven Stadnicki
    8 hours ago






  • 2




    $begingroup$
    That is a family of well-ordered sets, not a well-ordered family of (arbitrary sets).
    $endgroup$
    – Mike Battaglia
    8 hours ago










  • $begingroup$
    Ahh, I missed that distinction. Thank you!
    $endgroup$
    – Steven Stadnicki
    8 hours ago










  • $begingroup$
    I've never seen this axiom before. Does the family itself need to be a set or can it be a proper class?
    $endgroup$
    – Robert Shore
    8 hours ago










  • $begingroup$
    Google suggests this: settheory.mathtalks.org/andreas-blass-well-ordered-choice
    $endgroup$
    – Carl Mummert
    8 hours ago
















  • $begingroup$
    ...isn't this axiom a consequence of ZF? One can choose the (well-defined, since it's well-ordered) lexicographically least element of each set in the family...
    $endgroup$
    – Steven Stadnicki
    8 hours ago






  • 2




    $begingroup$
    That is a family of well-ordered sets, not a well-ordered family of (arbitrary sets).
    $endgroup$
    – Mike Battaglia
    8 hours ago










  • $begingroup$
    Ahh, I missed that distinction. Thank you!
    $endgroup$
    – Steven Stadnicki
    8 hours ago










  • $begingroup$
    I've never seen this axiom before. Does the family itself need to be a set or can it be a proper class?
    $endgroup$
    – Robert Shore
    8 hours ago










  • $begingroup$
    Google suggests this: settheory.mathtalks.org/andreas-blass-well-ordered-choice
    $endgroup$
    – Carl Mummert
    8 hours ago















$begingroup$
...isn't this axiom a consequence of ZF? One can choose the (well-defined, since it's well-ordered) lexicographically least element of each set in the family...
$endgroup$
– Steven Stadnicki
8 hours ago




$begingroup$
...isn't this axiom a consequence of ZF? One can choose the (well-defined, since it's well-ordered) lexicographically least element of each set in the family...
$endgroup$
– Steven Stadnicki
8 hours ago




2




2




$begingroup$
That is a family of well-ordered sets, not a well-ordered family of (arbitrary sets).
$endgroup$
– Mike Battaglia
8 hours ago




$begingroup$
That is a family of well-ordered sets, not a well-ordered family of (arbitrary sets).
$endgroup$
– Mike Battaglia
8 hours ago












$begingroup$
Ahh, I missed that distinction. Thank you!
$endgroup$
– Steven Stadnicki
8 hours ago




$begingroup$
Ahh, I missed that distinction. Thank you!
$endgroup$
– Steven Stadnicki
8 hours ago












$begingroup$
I've never seen this axiom before. Does the family itself need to be a set or can it be a proper class?
$endgroup$
– Robert Shore
8 hours ago




$begingroup$
I've never seen this axiom before. Does the family itself need to be a set or can it be a proper class?
$endgroup$
– Robert Shore
8 hours ago












$begingroup$
Google suggests this: settheory.mathtalks.org/andreas-blass-well-ordered-choice
$endgroup$
– Carl Mummert
8 hours ago




$begingroup$
Google suggests this: settheory.mathtalks.org/andreas-blass-well-ordered-choice
$endgroup$
– Carl Mummert
8 hours ago










1 Answer
1






active

oldest

votes


















6












$begingroup$

The axiom of well-ordered choice, or $sf AC_rm WO$, is strictly weaker than the axiom of choice itself. If we start with $L$ and add $omega_1$ Cohen reals, then go to $L(Bbb R)$, one can show that $sf AC_rm WO$ holds, while $Bbb R$ cannot be well-ordered there.



Pincus proved in the 1970s that this is equivalent to the following statement on Hartogs and Lindenbaum numbers:




$sf AC_rm WO$ is equivalent to the statement $forall x.aleph(x)=aleph^*(x)$.




Here, the Lindenbaum number, $aleph^*(x)$, is the least ordinal which $x$ cannot be mapped onto. One obvious fact is that $aleph(x)leqaleph^*(x)$.



In the late 1950s or early 1960s Jensen proved that this assumption also implies $sf DC$. This is also a very clever proof.



The conjunction of these two consequences gives us that $aleph_1leq 2^aleph_0$, as a result of a theorem of Shelah from the 1980s, this implies there is a non-measurable set of reals.



As far as Hahn–Banach, or other things of that sort, I do not believe that much is known on the topic. But to sum up, this axiom does not imply that the reals are well-ordered, but it does imply there is a non-measurable set of reals because there is a set of reals of size $aleph_1$ and $sf DC$ holds. Moreover, it is equivalent to saying that the Hartogs and Lindenbaum numbers are equal for all sets.






share|cite|improve this answer









$endgroup$












  • $begingroup$
    Do you have a source for a proof of $sf AC_rm WOiff forall x(aleph(x)=aleph^*(x))$?
    $endgroup$
    – Holo
    6 hours ago







  • 1




    $begingroup$
    @Holo karagila.org/2014/on-the-partition-principle
    $endgroup$
    – Asaf Karagila
    5 hours ago











Your Answer





StackExchange.ifUsing("editor", function ()
return StackExchange.using("mathjaxEditing", function ()
StackExchange.MarkdownEditor.creationCallbacks.add(function (editor, postfix)
StackExchange.mathjaxEditing.prepareWmdForMathJax(editor, postfix, [["$", "$"], ["\\(","\\)"]]);
);
);
, "mathjax-editing");

StackExchange.ready(function()
var channelOptions =
tags: "".split(" "),
id: "69"
;
initTagRenderer("".split(" "), "".split(" "), channelOptions);

StackExchange.using("externalEditor", function()
// Have to fire editor after snippets, if snippets enabled
if (StackExchange.settings.snippets.snippetsEnabled)
StackExchange.using("snippets", function()
createEditor();
);

else
createEditor();

);

function createEditor()
StackExchange.prepareEditor(
heartbeatType: 'answer',
autoActivateHeartbeat: false,
convertImagesToLinks: true,
noModals: true,
showLowRepImageUploadWarning: true,
reputationToPostImages: 10,
bindNavPrevention: true,
postfix: "",
imageUploader:
brandingHtml: "Powered by u003ca class="icon-imgur-white" href="https://imgur.com/"u003eu003c/au003e",
contentPolicyHtml: "User contributions licensed under u003ca href="https://creativecommons.org/licenses/by-sa/3.0/"u003ecc by-sa 3.0 with attribution requiredu003c/au003e u003ca href="https://stackoverflow.com/legal/content-policy"u003e(content policy)u003c/au003e",
allowUrls: true
,
noCode: true, onDemand: true,
discardSelector: ".discard-answer"
,immediatelyShowMarkdownHelp:true
);



);













draft saved

draft discarded


















StackExchange.ready(
function ()
StackExchange.openid.initPostLogin('.new-post-login', 'https%3a%2f%2fmath.stackexchange.com%2fquestions%2f3140508%2fhow-strong-is-the-axiom-of-well-ordered-choice%23new-answer', 'question_page');

);

Post as a guest















Required, but never shown

























1 Answer
1






active

oldest

votes








1 Answer
1






active

oldest

votes









active

oldest

votes






active

oldest

votes









6












$begingroup$

The axiom of well-ordered choice, or $sf AC_rm WO$, is strictly weaker than the axiom of choice itself. If we start with $L$ and add $omega_1$ Cohen reals, then go to $L(Bbb R)$, one can show that $sf AC_rm WO$ holds, while $Bbb R$ cannot be well-ordered there.



Pincus proved in the 1970s that this is equivalent to the following statement on Hartogs and Lindenbaum numbers:




$sf AC_rm WO$ is equivalent to the statement $forall x.aleph(x)=aleph^*(x)$.




Here, the Lindenbaum number, $aleph^*(x)$, is the least ordinal which $x$ cannot be mapped onto. One obvious fact is that $aleph(x)leqaleph^*(x)$.



In the late 1950s or early 1960s Jensen proved that this assumption also implies $sf DC$. This is also a very clever proof.



The conjunction of these two consequences gives us that $aleph_1leq 2^aleph_0$, as a result of a theorem of Shelah from the 1980s, this implies there is a non-measurable set of reals.



As far as Hahn–Banach, or other things of that sort, I do not believe that much is known on the topic. But to sum up, this axiom does not imply that the reals are well-ordered, but it does imply there is a non-measurable set of reals because there is a set of reals of size $aleph_1$ and $sf DC$ holds. Moreover, it is equivalent to saying that the Hartogs and Lindenbaum numbers are equal for all sets.






share|cite|improve this answer









$endgroup$












  • $begingroup$
    Do you have a source for a proof of $sf AC_rm WOiff forall x(aleph(x)=aleph^*(x))$?
    $endgroup$
    – Holo
    6 hours ago







  • 1




    $begingroup$
    @Holo karagila.org/2014/on-the-partition-principle
    $endgroup$
    – Asaf Karagila
    5 hours ago
















6












$begingroup$

The axiom of well-ordered choice, or $sf AC_rm WO$, is strictly weaker than the axiom of choice itself. If we start with $L$ and add $omega_1$ Cohen reals, then go to $L(Bbb R)$, one can show that $sf AC_rm WO$ holds, while $Bbb R$ cannot be well-ordered there.



Pincus proved in the 1970s that this is equivalent to the following statement on Hartogs and Lindenbaum numbers:




$sf AC_rm WO$ is equivalent to the statement $forall x.aleph(x)=aleph^*(x)$.




Here, the Lindenbaum number, $aleph^*(x)$, is the least ordinal which $x$ cannot be mapped onto. One obvious fact is that $aleph(x)leqaleph^*(x)$.



In the late 1950s or early 1960s Jensen proved that this assumption also implies $sf DC$. This is also a very clever proof.



The conjunction of these two consequences gives us that $aleph_1leq 2^aleph_0$, as a result of a theorem of Shelah from the 1980s, this implies there is a non-measurable set of reals.



As far as Hahn–Banach, or other things of that sort, I do not believe that much is known on the topic. But to sum up, this axiom does not imply that the reals are well-ordered, but it does imply there is a non-measurable set of reals because there is a set of reals of size $aleph_1$ and $sf DC$ holds. Moreover, it is equivalent to saying that the Hartogs and Lindenbaum numbers are equal for all sets.






share|cite|improve this answer









$endgroup$












  • $begingroup$
    Do you have a source for a proof of $sf AC_rm WOiff forall x(aleph(x)=aleph^*(x))$?
    $endgroup$
    – Holo
    6 hours ago







  • 1




    $begingroup$
    @Holo karagila.org/2014/on-the-partition-principle
    $endgroup$
    – Asaf Karagila
    5 hours ago














6












6








6





$begingroup$

The axiom of well-ordered choice, or $sf AC_rm WO$, is strictly weaker than the axiom of choice itself. If we start with $L$ and add $omega_1$ Cohen reals, then go to $L(Bbb R)$, one can show that $sf AC_rm WO$ holds, while $Bbb R$ cannot be well-ordered there.



Pincus proved in the 1970s that this is equivalent to the following statement on Hartogs and Lindenbaum numbers:




$sf AC_rm WO$ is equivalent to the statement $forall x.aleph(x)=aleph^*(x)$.




Here, the Lindenbaum number, $aleph^*(x)$, is the least ordinal which $x$ cannot be mapped onto. One obvious fact is that $aleph(x)leqaleph^*(x)$.



In the late 1950s or early 1960s Jensen proved that this assumption also implies $sf DC$. This is also a very clever proof.



The conjunction of these two consequences gives us that $aleph_1leq 2^aleph_0$, as a result of a theorem of Shelah from the 1980s, this implies there is a non-measurable set of reals.



As far as Hahn–Banach, or other things of that sort, I do not believe that much is known on the topic. But to sum up, this axiom does not imply that the reals are well-ordered, but it does imply there is a non-measurable set of reals because there is a set of reals of size $aleph_1$ and $sf DC$ holds. Moreover, it is equivalent to saying that the Hartogs and Lindenbaum numbers are equal for all sets.






share|cite|improve this answer









$endgroup$



The axiom of well-ordered choice, or $sf AC_rm WO$, is strictly weaker than the axiom of choice itself. If we start with $L$ and add $omega_1$ Cohen reals, then go to $L(Bbb R)$, one can show that $sf AC_rm WO$ holds, while $Bbb R$ cannot be well-ordered there.



Pincus proved in the 1970s that this is equivalent to the following statement on Hartogs and Lindenbaum numbers:




$sf AC_rm WO$ is equivalent to the statement $forall x.aleph(x)=aleph^*(x)$.




Here, the Lindenbaum number, $aleph^*(x)$, is the least ordinal which $x$ cannot be mapped onto. One obvious fact is that $aleph(x)leqaleph^*(x)$.



In the late 1950s or early 1960s Jensen proved that this assumption also implies $sf DC$. This is also a very clever proof.



The conjunction of these two consequences gives us that $aleph_1leq 2^aleph_0$, as a result of a theorem of Shelah from the 1980s, this implies there is a non-measurable set of reals.



As far as Hahn–Banach, or other things of that sort, I do not believe that much is known on the topic. But to sum up, this axiom does not imply that the reals are well-ordered, but it does imply there is a non-measurable set of reals because there is a set of reals of size $aleph_1$ and $sf DC$ holds. Moreover, it is equivalent to saying that the Hartogs and Lindenbaum numbers are equal for all sets.







share|cite|improve this answer












share|cite|improve this answer



share|cite|improve this answer










answered 7 hours ago









Asaf KaragilaAsaf Karagila

306k33437768




306k33437768











  • $begingroup$
    Do you have a source for a proof of $sf AC_rm WOiff forall x(aleph(x)=aleph^*(x))$?
    $endgroup$
    – Holo
    6 hours ago







  • 1




    $begingroup$
    @Holo karagila.org/2014/on-the-partition-principle
    $endgroup$
    – Asaf Karagila
    5 hours ago

















  • $begingroup$
    Do you have a source for a proof of $sf AC_rm WOiff forall x(aleph(x)=aleph^*(x))$?
    $endgroup$
    – Holo
    6 hours ago







  • 1




    $begingroup$
    @Holo karagila.org/2014/on-the-partition-principle
    $endgroup$
    – Asaf Karagila
    5 hours ago
















$begingroup$
Do you have a source for a proof of $sf AC_rm WOiff forall x(aleph(x)=aleph^*(x))$?
$endgroup$
– Holo
6 hours ago





$begingroup$
Do you have a source for a proof of $sf AC_rm WOiff forall x(aleph(x)=aleph^*(x))$?
$endgroup$
– Holo
6 hours ago





1




1




$begingroup$
@Holo karagila.org/2014/on-the-partition-principle
$endgroup$
– Asaf Karagila
5 hours ago





$begingroup$
@Holo karagila.org/2014/on-the-partition-principle
$endgroup$
– Asaf Karagila
5 hours ago


















draft saved

draft discarded
















































Thanks for contributing an answer to Mathematics Stack Exchange!


  • Please be sure to answer the question. Provide details and share your research!

But avoid


  • Asking for help, clarification, or responding to other answers.

  • Making statements based on opinion; back them up with references or personal experience.

Use MathJax to format equations. MathJax reference.


To learn more, see our tips on writing great answers.




draft saved


draft discarded














StackExchange.ready(
function ()
StackExchange.openid.initPostLogin('.new-post-login', 'https%3a%2f%2fmath.stackexchange.com%2fquestions%2f3140508%2fhow-strong-is-the-axiom-of-well-ordered-choice%23new-answer', 'question_page');

);

Post as a guest















Required, but never shown





















































Required, but never shown














Required, but never shown












Required, but never shown







Required, but never shown

































Required, but never shown














Required, but never shown












Required, but never shown







Required, but never shown







Popular posts from this blog

Mobil Contents History Mobil brands Former Mobil brands Lukoil transaction Mobil UK Mobil Australia Mobil New Zealand Mobil Greece Mobil in Japan Mobil in Canada Mobil Egypt See also References External links Navigation menuwww.mobil.com"Mobil Corporation"the original"Our Houston campus""Business & Finance: Socony-Vacuum Corp.""Popular Mechanics""Lubrite Technologies""Exxon Mobil campus 'clearly happening'""Toledo Blade - Google News Archive Search""The Lion and the Moose - How 2 Executives Pulled off the Biggest Merger Ever""ExxonMobil Press Release""Lubricants""Archived copy"the original"Mobil 1™ and Mobil Super™ motor oil and synthetic motor oil - Mobil™ Motor Oils""Mobil Delvac""Mobil Industrial website""The State of Competition in Gasoline Marketing: The Effects of Refiner Operations at Retail""Mobil Travel Guide to become Forbes Travel Guide""Hotel Rankings: Forbes Merges with Mobil"the original"Jamieson oil industry history""Mobil news""Caltex pumps for control""Watchdog blocks Caltex bid""Exxon Mobil sells service station network""Mobil Oil New Zealand Limited is New Zealand's oldest oil company, with predecessor companies having first established a presence in the country in 1896""ExxonMobil subsidiaries have a business history in New Zealand stretching back more than 120 years. We are involved in petroleum refining and distribution and the marketing of fuels, lubricants and chemical products""Archived copy"the original"Exxon Mobil to Sell Its Japanese Arm for $3.9 Billion""Gas station merger will end Esso and Mobil's long run in Japan""Esso moves to affiliate itself with PC Optimum, no longer Aeroplan, in loyalty point switch""Mobil brand of gas stations to launch in Canada after deal for 213 Loblaws-owned locations""Mobil Nears Completion of Rebranding 200 Loblaw Gas Stations""Learn about ExxonMobil's operations in Egypt""Petrol and Diesel Service Stations in Egypt - Mobil"Official websiteExxon Mobil corporate websiteMobil Industrial official websiteeeeeeeeDA04275022275790-40000 0001 0860 5061n82045453134887257134887257

Frič See also Navigation menuinternal link

Identify plant with long narrow paired leaves and reddish stems Planned maintenance scheduled April 17/18, 2019 at 00:00UTC (8:00pm US/Eastern) Announcing the arrival of Valued Associate #679: Cesar Manara Unicorn Meta Zoo #1: Why another podcast?What is this plant with long sharp leaves? Is it a weed?What is this 3ft high, stalky plant, with mid sized narrow leaves?What is this young shrub with opposite ovate, crenate leaves and reddish stems?What is this plant with large broad serrated leaves?Identify this upright branching weed with long leaves and reddish stemsPlease help me identify this bulbous plant with long, broad leaves and white flowersWhat is this small annual with narrow gray/green leaves and rust colored daisy-type flowers?What is this chilli plant?Does anyone know what type of chilli plant this is?Help identify this plant